Proving that the elements of a sequence will always be co-prime to each other.












2












$begingroup$


We are given the sequence $k$n = 6$^{{({2}^n)}}$ + 1. We must prove that the elements of this sequence are pairwise co-prime, i.e prove that if m $neq$ n then $gcd$($k$m,$k$n) = $1$.



I have proved that $k$n | ($k$n+1 - $2$) however I can't seem to extend this proof in order to prove every element is co prime.



All help would be greatly appreciated, cheers.










share|cite|improve this question









$endgroup$












  • $begingroup$
    What you have already shows you that $k_n$ and $k_{n+1}$ are coprime: the only possible common divisor would be $2$, but they're all odd. Could you do something similar for $k_{n}$ and $k_{n+2}$? Could something like this generalise?
    $endgroup$
    – user3482749
    Jan 31 at 16:41
















2












$begingroup$


We are given the sequence $k$n = 6$^{{({2}^n)}}$ + 1. We must prove that the elements of this sequence are pairwise co-prime, i.e prove that if m $neq$ n then $gcd$($k$m,$k$n) = $1$.



I have proved that $k$n | ($k$n+1 - $2$) however I can't seem to extend this proof in order to prove every element is co prime.



All help would be greatly appreciated, cheers.










share|cite|improve this question









$endgroup$












  • $begingroup$
    What you have already shows you that $k_n$ and $k_{n+1}$ are coprime: the only possible common divisor would be $2$, but they're all odd. Could you do something similar for $k_{n}$ and $k_{n+2}$? Could something like this generalise?
    $endgroup$
    – user3482749
    Jan 31 at 16:41














2












2








2





$begingroup$


We are given the sequence $k$n = 6$^{{({2}^n)}}$ + 1. We must prove that the elements of this sequence are pairwise co-prime, i.e prove that if m $neq$ n then $gcd$($k$m,$k$n) = $1$.



I have proved that $k$n | ($k$n+1 - $2$) however I can't seem to extend this proof in order to prove every element is co prime.



All help would be greatly appreciated, cheers.










share|cite|improve this question









$endgroup$




We are given the sequence $k$n = 6$^{{({2}^n)}}$ + 1. We must prove that the elements of this sequence are pairwise co-prime, i.e prove that if m $neq$ n then $gcd$($k$m,$k$n) = $1$.



I have proved that $k$n | ($k$n+1 - $2$) however I can't seem to extend this proof in order to prove every element is co prime.



All help would be greatly appreciated, cheers.







sequences-and-series number-theory elementary-number-theory proof-writing coprime






share|cite|improve this question













share|cite|improve this question











share|cite|improve this question




share|cite|improve this question










asked Jan 31 at 16:35







user637295



















  • $begingroup$
    What you have already shows you that $k_n$ and $k_{n+1}$ are coprime: the only possible common divisor would be $2$, but they're all odd. Could you do something similar for $k_{n}$ and $k_{n+2}$? Could something like this generalise?
    $endgroup$
    – user3482749
    Jan 31 at 16:41


















  • $begingroup$
    What you have already shows you that $k_n$ and $k_{n+1}$ are coprime: the only possible common divisor would be $2$, but they're all odd. Could you do something similar for $k_{n}$ and $k_{n+2}$? Could something like this generalise?
    $endgroup$
    – user3482749
    Jan 31 at 16:41
















$begingroup$
What you have already shows you that $k_n$ and $k_{n+1}$ are coprime: the only possible common divisor would be $2$, but they're all odd. Could you do something similar for $k_{n}$ and $k_{n+2}$? Could something like this generalise?
$endgroup$
– user3482749
Jan 31 at 16:41




$begingroup$
What you have already shows you that $k_n$ and $k_{n+1}$ are coprime: the only possible common divisor would be $2$, but they're all odd. Could you do something similar for $k_{n}$ and $k_{n+2}$? Could something like this generalise?
$endgroup$
– user3482749
Jan 31 at 16:41










3 Answers
3






active

oldest

votes


















4












$begingroup$

Claim: $$boxed {5prod_{i=0}^nk_i = k_{n+1}-2}$$



Pf: Consider the product $$P_n=prod_{i=0}^nk_i$$



Since $5=6^{(2^0)}-1$ we note that $$5P_n=left(6^{(2^0)}-1right)times left(6^{(2^0)}+1right)times prod_{i=1}^nk_i =left(6^{(2^1)}-1right)times left(6^{(2^1)}+1right)times prod_{i=2}^nk_i=$$ $$=left(6^{(2^2)}-1right)times left(6^{(2^2)}+1right)times prod_{i=3}^nk_i$$



Continuing in this way we see that $$5P_n=6^{(2^{n+1})}-1=k_{n+1}-2$$
as desired.



It follows that any common divisor of two of the $k_i$ would have to be a divisor of $2$. As all the $k_i$ are odd, we are done.



Note: since the point was raised in the comments, let me elaborate on the final paragraph. Suppose $i<j$. We wish to prove that $gcd(k_i,k_j)=1$. But $i<jimplies i≤j-1implies k_i,|,P_{j-1}$ Thus, $k_{i},|,5P_{j-1}=k_j-2$ Thus any common divisor of $k_i,k_j$ would have to divide $2$.






share|cite|improve this answer











$endgroup$













  • $begingroup$
    I did have to think a bit more about the last line to see it myself. I do see it now though. If the $k_n$s were not pairwise coprime then there exists an $n$ such that $k_{n+1}$ shares a common factor with $k_l$ for some $l<n+1$. If $k_{n+1}$ shares any common factors with any $k_l$; $l<n+1$, then $k_{n+1}$ shares a common factor with $5P_n$. But $5P_n +2 = k_{n+1}$, so the only factor $k_{n+1}$ could share with $5P_n$ is 2, and as $k_{n+1}$ is odd, it does not even share a factor of 2. Good answer!
    $endgroup$
    – Mike
    Jan 31 at 18:17








  • 1




    $begingroup$
    @Mike Yes, that's right. If $i<j$ then $k_i,|,5P_{j-1}=k_j-2$
    $endgroup$
    – lulu
    Jan 31 at 18:18












  • $begingroup$
    @lulu is there any way I could then use this result to then prove that there are infinitely many primes combined with the fundamental theorem of arithmetic?
    $endgroup$
    – user637295
    Jan 31 at 18:55










  • $begingroup$
    @LittleRichard Sure...let $p_i$ denote the least prime dividing $k_i$. Then all of the ${p_i}$ are distinct.
    $endgroup$
    – lulu
    Jan 31 at 18:58



















0












$begingroup$

If $p|k_n$, then $p|(k_{n+1}-2)$. If also $p|k_{n+1}$, then $$p|big(k_{n+1}-(k_{n+1}-2)big)$$
or $p|2$. Hence the only prime that could divide both $k_n$ and $k_{n+1}$ is $2$. However all the terms are odd, so $gcd(k_n,k_{n+1})=1$.



Now, you need a similar relationship between $k_n$ and $k_{n+m}$.






share|cite|improve this answer









$endgroup$













  • $begingroup$
    So would I just run through the same argument but instead of using k$_{n+1}$Just use k$_m$ ?
    $endgroup$
    – user637295
    Jan 31 at 19:07












  • $begingroup$
    You will first need to prove an analogue of $k_n|(k_{n+1}-2)$, but with $k_{n+m}$ instead of $k_{n+1}$.
    $endgroup$
    – vadim123
    Jan 31 at 19:53












  • $begingroup$
    so would I literally just use $k_{n+m}$ in replace of $k_{n+1}$ ?
    $endgroup$
    – user637295
    Feb 1 at 17:05










  • $begingroup$
    I'm not 100% sure how I can prove the k$_{n+m}$ case, is there any way you could help? Cheers
    $endgroup$
    – user637295
    Feb 3 at 17:39



















0












$begingroup$

note that the expression can be mod $$6(2^m)+1,quad m<n$$ this gives $$6(2^n bmod 6(2^m)+1)+1$$ if the mod is $2^m$ then $2^n$ is different from a value we know divides by it ( the number itself) by a multiple of the moduli itself. That implies $$2^n=(6(2^m)+1)k+2^m$$ dividing boths sides by $2^m$ gives $$2^{n-m}=6k+k=7k$$, since 7 divides no power of 2 it then follows the mod can't hold, which makes the division by divisors of $6(2^m)+1$ also not work unless they can divide a power of 2 ( which being odd they can't)






share|cite|improve this answer











$endgroup$














    Your Answer





    StackExchange.ifUsing("editor", function () {
    return StackExchange.using("mathjaxEditing", function () {
    StackExchange.MarkdownEditor.creationCallbacks.add(function (editor, postfix) {
    StackExchange.mathjaxEditing.prepareWmdForMathJax(editor, postfix, [["$", "$"], ["\\(","\\)"]]);
    });
    });
    }, "mathjax-editing");

    StackExchange.ready(function() {
    var channelOptions = {
    tags: "".split(" "),
    id: "69"
    };
    initTagRenderer("".split(" "), "".split(" "), channelOptions);

    StackExchange.using("externalEditor", function() {
    // Have to fire editor after snippets, if snippets enabled
    if (StackExchange.settings.snippets.snippetsEnabled) {
    StackExchange.using("snippets", function() {
    createEditor();
    });
    }
    else {
    createEditor();
    }
    });

    function createEditor() {
    StackExchange.prepareEditor({
    heartbeatType: 'answer',
    autoActivateHeartbeat: false,
    convertImagesToLinks: true,
    noModals: true,
    showLowRepImageUploadWarning: true,
    reputationToPostImages: 10,
    bindNavPrevention: true,
    postfix: "",
    imageUploader: {
    brandingHtml: "Powered by u003ca class="icon-imgur-white" href="https://imgur.com/"u003eu003c/au003e",
    contentPolicyHtml: "User contributions licensed under u003ca href="https://creativecommons.org/licenses/by-sa/3.0/"u003ecc by-sa 3.0 with attribution requiredu003c/au003e u003ca href="https://stackoverflow.com/legal/content-policy"u003e(content policy)u003c/au003e",
    allowUrls: true
    },
    noCode: true, onDemand: true,
    discardSelector: ".discard-answer"
    ,immediatelyShowMarkdownHelp:true
    });


    }
    });














    draft saved

    draft discarded


















    StackExchange.ready(
    function () {
    StackExchange.openid.initPostLogin('.new-post-login', 'https%3a%2f%2fmath.stackexchange.com%2fquestions%2f3095107%2fproving-that-the-elements-of-a-sequence-will-always-be-co-prime-to-each-other%23new-answer', 'question_page');
    }
    );

    Post as a guest















    Required, but never shown
























    3 Answers
    3






    active

    oldest

    votes








    3 Answers
    3






    active

    oldest

    votes









    active

    oldest

    votes






    active

    oldest

    votes









    4












    $begingroup$

    Claim: $$boxed {5prod_{i=0}^nk_i = k_{n+1}-2}$$



    Pf: Consider the product $$P_n=prod_{i=0}^nk_i$$



    Since $5=6^{(2^0)}-1$ we note that $$5P_n=left(6^{(2^0)}-1right)times left(6^{(2^0)}+1right)times prod_{i=1}^nk_i =left(6^{(2^1)}-1right)times left(6^{(2^1)}+1right)times prod_{i=2}^nk_i=$$ $$=left(6^{(2^2)}-1right)times left(6^{(2^2)}+1right)times prod_{i=3}^nk_i$$



    Continuing in this way we see that $$5P_n=6^{(2^{n+1})}-1=k_{n+1}-2$$
    as desired.



    It follows that any common divisor of two of the $k_i$ would have to be a divisor of $2$. As all the $k_i$ are odd, we are done.



    Note: since the point was raised in the comments, let me elaborate on the final paragraph. Suppose $i<j$. We wish to prove that $gcd(k_i,k_j)=1$. But $i<jimplies i≤j-1implies k_i,|,P_{j-1}$ Thus, $k_{i},|,5P_{j-1}=k_j-2$ Thus any common divisor of $k_i,k_j$ would have to divide $2$.






    share|cite|improve this answer











    $endgroup$













    • $begingroup$
      I did have to think a bit more about the last line to see it myself. I do see it now though. If the $k_n$s were not pairwise coprime then there exists an $n$ such that $k_{n+1}$ shares a common factor with $k_l$ for some $l<n+1$. If $k_{n+1}$ shares any common factors with any $k_l$; $l<n+1$, then $k_{n+1}$ shares a common factor with $5P_n$. But $5P_n +2 = k_{n+1}$, so the only factor $k_{n+1}$ could share with $5P_n$ is 2, and as $k_{n+1}$ is odd, it does not even share a factor of 2. Good answer!
      $endgroup$
      – Mike
      Jan 31 at 18:17








    • 1




      $begingroup$
      @Mike Yes, that's right. If $i<j$ then $k_i,|,5P_{j-1}=k_j-2$
      $endgroup$
      – lulu
      Jan 31 at 18:18












    • $begingroup$
      @lulu is there any way I could then use this result to then prove that there are infinitely many primes combined with the fundamental theorem of arithmetic?
      $endgroup$
      – user637295
      Jan 31 at 18:55










    • $begingroup$
      @LittleRichard Sure...let $p_i$ denote the least prime dividing $k_i$. Then all of the ${p_i}$ are distinct.
      $endgroup$
      – lulu
      Jan 31 at 18:58
















    4












    $begingroup$

    Claim: $$boxed {5prod_{i=0}^nk_i = k_{n+1}-2}$$



    Pf: Consider the product $$P_n=prod_{i=0}^nk_i$$



    Since $5=6^{(2^0)}-1$ we note that $$5P_n=left(6^{(2^0)}-1right)times left(6^{(2^0)}+1right)times prod_{i=1}^nk_i =left(6^{(2^1)}-1right)times left(6^{(2^1)}+1right)times prod_{i=2}^nk_i=$$ $$=left(6^{(2^2)}-1right)times left(6^{(2^2)}+1right)times prod_{i=3}^nk_i$$



    Continuing in this way we see that $$5P_n=6^{(2^{n+1})}-1=k_{n+1}-2$$
    as desired.



    It follows that any common divisor of two of the $k_i$ would have to be a divisor of $2$. As all the $k_i$ are odd, we are done.



    Note: since the point was raised in the comments, let me elaborate on the final paragraph. Suppose $i<j$. We wish to prove that $gcd(k_i,k_j)=1$. But $i<jimplies i≤j-1implies k_i,|,P_{j-1}$ Thus, $k_{i},|,5P_{j-1}=k_j-2$ Thus any common divisor of $k_i,k_j$ would have to divide $2$.






    share|cite|improve this answer











    $endgroup$













    • $begingroup$
      I did have to think a bit more about the last line to see it myself. I do see it now though. If the $k_n$s were not pairwise coprime then there exists an $n$ such that $k_{n+1}$ shares a common factor with $k_l$ for some $l<n+1$. If $k_{n+1}$ shares any common factors with any $k_l$; $l<n+1$, then $k_{n+1}$ shares a common factor with $5P_n$. But $5P_n +2 = k_{n+1}$, so the only factor $k_{n+1}$ could share with $5P_n$ is 2, and as $k_{n+1}$ is odd, it does not even share a factor of 2. Good answer!
      $endgroup$
      – Mike
      Jan 31 at 18:17








    • 1




      $begingroup$
      @Mike Yes, that's right. If $i<j$ then $k_i,|,5P_{j-1}=k_j-2$
      $endgroup$
      – lulu
      Jan 31 at 18:18












    • $begingroup$
      @lulu is there any way I could then use this result to then prove that there are infinitely many primes combined with the fundamental theorem of arithmetic?
      $endgroup$
      – user637295
      Jan 31 at 18:55










    • $begingroup$
      @LittleRichard Sure...let $p_i$ denote the least prime dividing $k_i$. Then all of the ${p_i}$ are distinct.
      $endgroup$
      – lulu
      Jan 31 at 18:58














    4












    4








    4





    $begingroup$

    Claim: $$boxed {5prod_{i=0}^nk_i = k_{n+1}-2}$$



    Pf: Consider the product $$P_n=prod_{i=0}^nk_i$$



    Since $5=6^{(2^0)}-1$ we note that $$5P_n=left(6^{(2^0)}-1right)times left(6^{(2^0)}+1right)times prod_{i=1}^nk_i =left(6^{(2^1)}-1right)times left(6^{(2^1)}+1right)times prod_{i=2}^nk_i=$$ $$=left(6^{(2^2)}-1right)times left(6^{(2^2)}+1right)times prod_{i=3}^nk_i$$



    Continuing in this way we see that $$5P_n=6^{(2^{n+1})}-1=k_{n+1}-2$$
    as desired.



    It follows that any common divisor of two of the $k_i$ would have to be a divisor of $2$. As all the $k_i$ are odd, we are done.



    Note: since the point was raised in the comments, let me elaborate on the final paragraph. Suppose $i<j$. We wish to prove that $gcd(k_i,k_j)=1$. But $i<jimplies i≤j-1implies k_i,|,P_{j-1}$ Thus, $k_{i},|,5P_{j-1}=k_j-2$ Thus any common divisor of $k_i,k_j$ would have to divide $2$.






    share|cite|improve this answer











    $endgroup$



    Claim: $$boxed {5prod_{i=0}^nk_i = k_{n+1}-2}$$



    Pf: Consider the product $$P_n=prod_{i=0}^nk_i$$



    Since $5=6^{(2^0)}-1$ we note that $$5P_n=left(6^{(2^0)}-1right)times left(6^{(2^0)}+1right)times prod_{i=1}^nk_i =left(6^{(2^1)}-1right)times left(6^{(2^1)}+1right)times prod_{i=2}^nk_i=$$ $$=left(6^{(2^2)}-1right)times left(6^{(2^2)}+1right)times prod_{i=3}^nk_i$$



    Continuing in this way we see that $$5P_n=6^{(2^{n+1})}-1=k_{n+1}-2$$
    as desired.



    It follows that any common divisor of two of the $k_i$ would have to be a divisor of $2$. As all the $k_i$ are odd, we are done.



    Note: since the point was raised in the comments, let me elaborate on the final paragraph. Suppose $i<j$. We wish to prove that $gcd(k_i,k_j)=1$. But $i<jimplies i≤j-1implies k_i,|,P_{j-1}$ Thus, $k_{i},|,5P_{j-1}=k_j-2$ Thus any common divisor of $k_i,k_j$ would have to divide $2$.







    share|cite|improve this answer














    share|cite|improve this answer



    share|cite|improve this answer








    edited Jan 31 at 20:09

























    answered Jan 31 at 16:55









    lulululu

    43.6k25081




    43.6k25081












    • $begingroup$
      I did have to think a bit more about the last line to see it myself. I do see it now though. If the $k_n$s were not pairwise coprime then there exists an $n$ such that $k_{n+1}$ shares a common factor with $k_l$ for some $l<n+1$. If $k_{n+1}$ shares any common factors with any $k_l$; $l<n+1$, then $k_{n+1}$ shares a common factor with $5P_n$. But $5P_n +2 = k_{n+1}$, so the only factor $k_{n+1}$ could share with $5P_n$ is 2, and as $k_{n+1}$ is odd, it does not even share a factor of 2. Good answer!
      $endgroup$
      – Mike
      Jan 31 at 18:17








    • 1




      $begingroup$
      @Mike Yes, that's right. If $i<j$ then $k_i,|,5P_{j-1}=k_j-2$
      $endgroup$
      – lulu
      Jan 31 at 18:18












    • $begingroup$
      @lulu is there any way I could then use this result to then prove that there are infinitely many primes combined with the fundamental theorem of arithmetic?
      $endgroup$
      – user637295
      Jan 31 at 18:55










    • $begingroup$
      @LittleRichard Sure...let $p_i$ denote the least prime dividing $k_i$. Then all of the ${p_i}$ are distinct.
      $endgroup$
      – lulu
      Jan 31 at 18:58


















    • $begingroup$
      I did have to think a bit more about the last line to see it myself. I do see it now though. If the $k_n$s were not pairwise coprime then there exists an $n$ such that $k_{n+1}$ shares a common factor with $k_l$ for some $l<n+1$. If $k_{n+1}$ shares any common factors with any $k_l$; $l<n+1$, then $k_{n+1}$ shares a common factor with $5P_n$. But $5P_n +2 = k_{n+1}$, so the only factor $k_{n+1}$ could share with $5P_n$ is 2, and as $k_{n+1}$ is odd, it does not even share a factor of 2. Good answer!
      $endgroup$
      – Mike
      Jan 31 at 18:17








    • 1




      $begingroup$
      @Mike Yes, that's right. If $i<j$ then $k_i,|,5P_{j-1}=k_j-2$
      $endgroup$
      – lulu
      Jan 31 at 18:18












    • $begingroup$
      @lulu is there any way I could then use this result to then prove that there are infinitely many primes combined with the fundamental theorem of arithmetic?
      $endgroup$
      – user637295
      Jan 31 at 18:55










    • $begingroup$
      @LittleRichard Sure...let $p_i$ denote the least prime dividing $k_i$. Then all of the ${p_i}$ are distinct.
      $endgroup$
      – lulu
      Jan 31 at 18:58
















    $begingroup$
    I did have to think a bit more about the last line to see it myself. I do see it now though. If the $k_n$s were not pairwise coprime then there exists an $n$ such that $k_{n+1}$ shares a common factor with $k_l$ for some $l<n+1$. If $k_{n+1}$ shares any common factors with any $k_l$; $l<n+1$, then $k_{n+1}$ shares a common factor with $5P_n$. But $5P_n +2 = k_{n+1}$, so the only factor $k_{n+1}$ could share with $5P_n$ is 2, and as $k_{n+1}$ is odd, it does not even share a factor of 2. Good answer!
    $endgroup$
    – Mike
    Jan 31 at 18:17






    $begingroup$
    I did have to think a bit more about the last line to see it myself. I do see it now though. If the $k_n$s were not pairwise coprime then there exists an $n$ such that $k_{n+1}$ shares a common factor with $k_l$ for some $l<n+1$. If $k_{n+1}$ shares any common factors with any $k_l$; $l<n+1$, then $k_{n+1}$ shares a common factor with $5P_n$. But $5P_n +2 = k_{n+1}$, so the only factor $k_{n+1}$ could share with $5P_n$ is 2, and as $k_{n+1}$ is odd, it does not even share a factor of 2. Good answer!
    $endgroup$
    – Mike
    Jan 31 at 18:17






    1




    1




    $begingroup$
    @Mike Yes, that's right. If $i<j$ then $k_i,|,5P_{j-1}=k_j-2$
    $endgroup$
    – lulu
    Jan 31 at 18:18






    $begingroup$
    @Mike Yes, that's right. If $i<j$ then $k_i,|,5P_{j-1}=k_j-2$
    $endgroup$
    – lulu
    Jan 31 at 18:18














    $begingroup$
    @lulu is there any way I could then use this result to then prove that there are infinitely many primes combined with the fundamental theorem of arithmetic?
    $endgroup$
    – user637295
    Jan 31 at 18:55




    $begingroup$
    @lulu is there any way I could then use this result to then prove that there are infinitely many primes combined with the fundamental theorem of arithmetic?
    $endgroup$
    – user637295
    Jan 31 at 18:55












    $begingroup$
    @LittleRichard Sure...let $p_i$ denote the least prime dividing $k_i$. Then all of the ${p_i}$ are distinct.
    $endgroup$
    – lulu
    Jan 31 at 18:58




    $begingroup$
    @LittleRichard Sure...let $p_i$ denote the least prime dividing $k_i$. Then all of the ${p_i}$ are distinct.
    $endgroup$
    – lulu
    Jan 31 at 18:58











    0












    $begingroup$

    If $p|k_n$, then $p|(k_{n+1}-2)$. If also $p|k_{n+1}$, then $$p|big(k_{n+1}-(k_{n+1}-2)big)$$
    or $p|2$. Hence the only prime that could divide both $k_n$ and $k_{n+1}$ is $2$. However all the terms are odd, so $gcd(k_n,k_{n+1})=1$.



    Now, you need a similar relationship between $k_n$ and $k_{n+m}$.






    share|cite|improve this answer









    $endgroup$













    • $begingroup$
      So would I just run through the same argument but instead of using k$_{n+1}$Just use k$_m$ ?
      $endgroup$
      – user637295
      Jan 31 at 19:07












    • $begingroup$
      You will first need to prove an analogue of $k_n|(k_{n+1}-2)$, but with $k_{n+m}$ instead of $k_{n+1}$.
      $endgroup$
      – vadim123
      Jan 31 at 19:53












    • $begingroup$
      so would I literally just use $k_{n+m}$ in replace of $k_{n+1}$ ?
      $endgroup$
      – user637295
      Feb 1 at 17:05










    • $begingroup$
      I'm not 100% sure how I can prove the k$_{n+m}$ case, is there any way you could help? Cheers
      $endgroup$
      – user637295
      Feb 3 at 17:39
















    0












    $begingroup$

    If $p|k_n$, then $p|(k_{n+1}-2)$. If also $p|k_{n+1}$, then $$p|big(k_{n+1}-(k_{n+1}-2)big)$$
    or $p|2$. Hence the only prime that could divide both $k_n$ and $k_{n+1}$ is $2$. However all the terms are odd, so $gcd(k_n,k_{n+1})=1$.



    Now, you need a similar relationship between $k_n$ and $k_{n+m}$.






    share|cite|improve this answer









    $endgroup$













    • $begingroup$
      So would I just run through the same argument but instead of using k$_{n+1}$Just use k$_m$ ?
      $endgroup$
      – user637295
      Jan 31 at 19:07












    • $begingroup$
      You will first need to prove an analogue of $k_n|(k_{n+1}-2)$, but with $k_{n+m}$ instead of $k_{n+1}$.
      $endgroup$
      – vadim123
      Jan 31 at 19:53












    • $begingroup$
      so would I literally just use $k_{n+m}$ in replace of $k_{n+1}$ ?
      $endgroup$
      – user637295
      Feb 1 at 17:05










    • $begingroup$
      I'm not 100% sure how I can prove the k$_{n+m}$ case, is there any way you could help? Cheers
      $endgroup$
      – user637295
      Feb 3 at 17:39














    0












    0








    0





    $begingroup$

    If $p|k_n$, then $p|(k_{n+1}-2)$. If also $p|k_{n+1}$, then $$p|big(k_{n+1}-(k_{n+1}-2)big)$$
    or $p|2$. Hence the only prime that could divide both $k_n$ and $k_{n+1}$ is $2$. However all the terms are odd, so $gcd(k_n,k_{n+1})=1$.



    Now, you need a similar relationship between $k_n$ and $k_{n+m}$.






    share|cite|improve this answer









    $endgroup$



    If $p|k_n$, then $p|(k_{n+1}-2)$. If also $p|k_{n+1}$, then $$p|big(k_{n+1}-(k_{n+1}-2)big)$$
    or $p|2$. Hence the only prime that could divide both $k_n$ and $k_{n+1}$ is $2$. However all the terms are odd, so $gcd(k_n,k_{n+1})=1$.



    Now, you need a similar relationship between $k_n$ and $k_{n+m}$.







    share|cite|improve this answer












    share|cite|improve this answer



    share|cite|improve this answer










    answered Jan 31 at 16:42









    vadim123vadim123

    76.5k897191




    76.5k897191












    • $begingroup$
      So would I just run through the same argument but instead of using k$_{n+1}$Just use k$_m$ ?
      $endgroup$
      – user637295
      Jan 31 at 19:07












    • $begingroup$
      You will first need to prove an analogue of $k_n|(k_{n+1}-2)$, but with $k_{n+m}$ instead of $k_{n+1}$.
      $endgroup$
      – vadim123
      Jan 31 at 19:53












    • $begingroup$
      so would I literally just use $k_{n+m}$ in replace of $k_{n+1}$ ?
      $endgroup$
      – user637295
      Feb 1 at 17:05










    • $begingroup$
      I'm not 100% sure how I can prove the k$_{n+m}$ case, is there any way you could help? Cheers
      $endgroup$
      – user637295
      Feb 3 at 17:39


















    • $begingroup$
      So would I just run through the same argument but instead of using k$_{n+1}$Just use k$_m$ ?
      $endgroup$
      – user637295
      Jan 31 at 19:07












    • $begingroup$
      You will first need to prove an analogue of $k_n|(k_{n+1}-2)$, but with $k_{n+m}$ instead of $k_{n+1}$.
      $endgroup$
      – vadim123
      Jan 31 at 19:53












    • $begingroup$
      so would I literally just use $k_{n+m}$ in replace of $k_{n+1}$ ?
      $endgroup$
      – user637295
      Feb 1 at 17:05










    • $begingroup$
      I'm not 100% sure how I can prove the k$_{n+m}$ case, is there any way you could help? Cheers
      $endgroup$
      – user637295
      Feb 3 at 17:39
















    $begingroup$
    So would I just run through the same argument but instead of using k$_{n+1}$Just use k$_m$ ?
    $endgroup$
    – user637295
    Jan 31 at 19:07






    $begingroup$
    So would I just run through the same argument but instead of using k$_{n+1}$Just use k$_m$ ?
    $endgroup$
    – user637295
    Jan 31 at 19:07














    $begingroup$
    You will first need to prove an analogue of $k_n|(k_{n+1}-2)$, but with $k_{n+m}$ instead of $k_{n+1}$.
    $endgroup$
    – vadim123
    Jan 31 at 19:53






    $begingroup$
    You will first need to prove an analogue of $k_n|(k_{n+1}-2)$, but with $k_{n+m}$ instead of $k_{n+1}$.
    $endgroup$
    – vadim123
    Jan 31 at 19:53














    $begingroup$
    so would I literally just use $k_{n+m}$ in replace of $k_{n+1}$ ?
    $endgroup$
    – user637295
    Feb 1 at 17:05




    $begingroup$
    so would I literally just use $k_{n+m}$ in replace of $k_{n+1}$ ?
    $endgroup$
    – user637295
    Feb 1 at 17:05












    $begingroup$
    I'm not 100% sure how I can prove the k$_{n+m}$ case, is there any way you could help? Cheers
    $endgroup$
    – user637295
    Feb 3 at 17:39




    $begingroup$
    I'm not 100% sure how I can prove the k$_{n+m}$ case, is there any way you could help? Cheers
    $endgroup$
    – user637295
    Feb 3 at 17:39











    0












    $begingroup$

    note that the expression can be mod $$6(2^m)+1,quad m<n$$ this gives $$6(2^n bmod 6(2^m)+1)+1$$ if the mod is $2^m$ then $2^n$ is different from a value we know divides by it ( the number itself) by a multiple of the moduli itself. That implies $$2^n=(6(2^m)+1)k+2^m$$ dividing boths sides by $2^m$ gives $$2^{n-m}=6k+k=7k$$, since 7 divides no power of 2 it then follows the mod can't hold, which makes the division by divisors of $6(2^m)+1$ also not work unless they can divide a power of 2 ( which being odd they can't)






    share|cite|improve this answer











    $endgroup$


















      0












      $begingroup$

      note that the expression can be mod $$6(2^m)+1,quad m<n$$ this gives $$6(2^n bmod 6(2^m)+1)+1$$ if the mod is $2^m$ then $2^n$ is different from a value we know divides by it ( the number itself) by a multiple of the moduli itself. That implies $$2^n=(6(2^m)+1)k+2^m$$ dividing boths sides by $2^m$ gives $$2^{n-m}=6k+k=7k$$, since 7 divides no power of 2 it then follows the mod can't hold, which makes the division by divisors of $6(2^m)+1$ also not work unless they can divide a power of 2 ( which being odd they can't)






      share|cite|improve this answer











      $endgroup$
















        0












        0








        0





        $begingroup$

        note that the expression can be mod $$6(2^m)+1,quad m<n$$ this gives $$6(2^n bmod 6(2^m)+1)+1$$ if the mod is $2^m$ then $2^n$ is different from a value we know divides by it ( the number itself) by a multiple of the moduli itself. That implies $$2^n=(6(2^m)+1)k+2^m$$ dividing boths sides by $2^m$ gives $$2^{n-m}=6k+k=7k$$, since 7 divides no power of 2 it then follows the mod can't hold, which makes the division by divisors of $6(2^m)+1$ also not work unless they can divide a power of 2 ( which being odd they can't)






        share|cite|improve this answer











        $endgroup$



        note that the expression can be mod $$6(2^m)+1,quad m<n$$ this gives $$6(2^n bmod 6(2^m)+1)+1$$ if the mod is $2^m$ then $2^n$ is different from a value we know divides by it ( the number itself) by a multiple of the moduli itself. That implies $$2^n=(6(2^m)+1)k+2^m$$ dividing boths sides by $2^m$ gives $$2^{n-m}=6k+k=7k$$, since 7 divides no power of 2 it then follows the mod can't hold, which makes the division by divisors of $6(2^m)+1$ also not work unless they can divide a power of 2 ( which being odd they can't)







        share|cite|improve this answer














        share|cite|improve this answer



        share|cite|improve this answer








        edited Feb 23 at 16:46

























        answered Feb 23 at 16:40









        Roddy MacPheeRoddy MacPhee

        722118




        722118






























            draft saved

            draft discarded




















































            Thanks for contributing an answer to Mathematics Stack Exchange!


            • Please be sure to answer the question. Provide details and share your research!

            But avoid



            • Asking for help, clarification, or responding to other answers.

            • Making statements based on opinion; back them up with references or personal experience.


            Use MathJax to format equations. MathJax reference.


            To learn more, see our tips on writing great answers.




            draft saved


            draft discarded














            StackExchange.ready(
            function () {
            StackExchange.openid.initPostLogin('.new-post-login', 'https%3a%2f%2fmath.stackexchange.com%2fquestions%2f3095107%2fproving-that-the-elements-of-a-sequence-will-always-be-co-prime-to-each-other%23new-answer', 'question_page');
            }
            );

            Post as a guest















            Required, but never shown





















































            Required, but never shown














            Required, but never shown












            Required, but never shown







            Required, but never shown

































            Required, but never shown














            Required, but never shown












            Required, but never shown







            Required, but never shown







            Popular posts from this blog

            Can a sorcerer learn a 5th-level spell early by creating spell slots using the Font of Magic feature?

            Does disintegrating a polymorphed enemy still kill it after the 2018 errata?

            A Topological Invariant for $pi_3(U(n))$